Questions tagged [irrationality-measure]

The Liouville-Roth irrationality measure of a real number $x$ is a measure of how "closely" it can be approximated by rationals.

The Liouville-Roth irrationality measure (irrationality exponent, approximation exponent, or Liouville–Roth constant) of a real number $x$ is a measure of how "closely" it can be approximated by rationals. The largest possible value for $\mu$ such that $ 0<\left|x-{\frac {p}{q}}\right|<{\frac {1}{q^{\mu }}} $ is satisfied by an infinite number of integer pairs $(p, q)$ with $q > 0$ is defined to be the irrationality measure of $x$. For any value $\mu$ less than this upper bound, the infinite set of all rationals $\frac p q$ satisfying the above inequality yield an approximation of $x$. Conversely, if $\mu$ is greater than the upper bound, then there are at most finitely many $(p, q)$ with $q > 0$ that satisfy the inequality; thus, the opposite inequality holds for all larger values of $q$. In other words, given the irrationality measure $\mu$ of a real number $x$, whenever a rational approximation $x \approx \frac p q$, $p,q \in \mathbb Z$ and $q>0$, yields $n + 1$ exact decimal digits, we have

$$ \frac {1}{10^{n}}\ge \left|x-\frac {p}{q}\right|\ge \frac {1}{q^{\mu +\epsilon }} $$

for any $\epsilon>0$, except for at most a finite number of "lucky" pairs $(p, q)$.

For a rational number $\alpha$ the irrationality measure is $\mu(\alpha) = 1$. The Thue–Siegel–Roth theorem states that if $\alpha$ is an algebraic number, real but not rational, then $\mu(\alpha) = 2$.

Almost all numbers have an irrationality measure equal to $2$.

Transcendental numbers have irrationality measure $2$ or greater. For example, the transcendental number $e$ has $\mu(e) = 2$. The irrationality measures of $\pi$, $\log 2$, and $\log 3$ are at most $7.103205334137$, $3.57455391$, and $5.125$, respectively.

It has been proven that if the series $ \sum _{n=1}^{\infty }{\frac {\csc ^{2}n}{n^{3}}} $ (where $n$ is in radians) converges, then $ \pi $'s irrationality measure is at most $2.5$.

The Liouville numbers are precisely those numbers having infinite irrationality measure.

Source: Wikipedia

30 questions
24
votes
1 answer

Are all transcendental numbers a zero of a power series?

So I came across the concept of extending the notion of irrationality to higher degree polynomials. The base case of this is standard irrationality. That is, a number is irrational if it cannot be expressed as the ratio of two integers. This is…
20
votes
1 answer

Can multiples of two reals stay separated?

This question is inspired by my own answer to this question. For a real number $x > 0$, define $$ S(x) = \{\lfloor kx \rfloor \mid k \in \mathbb N\}. $$ Are there positive real numbers $x, y$ such that $S(x) \cap S(y)$ is finite? If $x, y$ are…
Mees de Vries
  • 24,889
  • 1
  • 40
  • 73
18
votes
1 answer

On the proximity of $a\sqrt b+b\sqrt a$ to an integer

Let $\Sigma$ denote all ordered pairs $(a,b)$ of positive, square-free integers with $a> b$. What is the infimum of all $\kappa$ such that $$\left\{(a,b) \in \Sigma:a\sqrt b+b\sqrt a\,\,\text{is within}\,\,\frac1{(ab)^{\kappa}}\,\,\text{of an…
TheSimpliFire
  • 25,185
  • 9
  • 47
  • 115
12
votes
1 answer

Recent progress in the irrationality measure of $\pi$

The context. For any real number $x$, let's define the quantity $$\mu(x):=\sup\left\{\mu\in\mathbb R_+\, \text{there is an infinity of $(p,q)\in\mathbb Z\times\mathbb N$ such that}\ 0<\left\vert x-\frac pq\right\vert<\frac 1{q^{\mu}}\right\},$$ and…
E. Joseph
  • 14,453
  • 8
  • 36
  • 67
10
votes
1 answer

Can the formula $\frac\pi2=(\frac21)^{1/2}(\frac{2^2}{1\cdot3})^{1/4}(\frac{2^3\cdot4}{1\cdot3^3})^{1/8}\cdots$ prove the irrationality of $\pi$?

A less known product formula for $\pi$, due to Sondow, is the following: $$ \frac{\pi}{2}= \left(\frac{2}{1}\right)^{1/2} \left(\frac{2^2}{1\cdot3}\right)^{1/4} \left(\frac{2^3\cdot4}{1\cdot3^3}\right)^{1/8}…
Klangen
  • 4,788
  • 5
  • 29
  • 64
9
votes
1 answer

Why can we not establish the irrationality of Catalan's constant the same way as $\zeta(3)$?

One of the main ingredients in Apéry's proof of the irrationality of $\zeta(3)$ is the existence of the fast-converging series: $$ {\displaystyle {\begin{aligned}\zeta (3)&={\frac {5}{2}}\sum _{k=1}^{\infty }{\frac {(-1)^{k-1}}{{\binom…
Klangen
  • 4,788
  • 5
  • 29
  • 64
8
votes
0 answers

Approximation of a random number with quadratic integers

Consider the following claim: Claim: Let $x$ be a real random variable distributed according to the uniform distribution on the unit interval $U(0,1)$. Then for any quadratic irrational number $\alpha$, and real $\epsilon>0$, there exist finite…
8
votes
1 answer

Does $\sum_{k=1}^n|\cot \sqrt2\pi k|$ tends to $An\ln n$ as $n\to\infty$?

Question: How can we prove that $$L(n)=\sum_{k=1}^n\left|\cot \sqrt2\pi k\right|=\Theta(n\log n)$$ as $n\to\infty$? Furthermore, if $\sqrt2$ is replaced with a quadratic irrational number, does it still holds? Numerical experiment. By plotting…
7
votes
1 answer

Applying the Beukers-like irrationality proof for $\zeta(2)$/$\zeta(3)$ to Catalan's Constant: Where does it fail?

Many people have tried and failed to extend Apery's Irrationality proof of $\zeta(3)$ to Catalan's constant, by looking for a fast converging series for Catalan's constant analogous to the one for $\zeta(3)$ that Apery utilized: $${\displaystyle…
3
votes
0 answers

Roth's theorem: contradiction?

Roth's theorem says that for irrational algebraic number $\alpha$ and $\epsilon>0$, there are finitely many solutions to this: $$\displaystyle \left|\alpha-\frac pq\right|<\frac 1{q^{2+\epsilon}}$$ but from what I understand from continued…
2
votes
1 answer

Measuring Irrationality à la 3Blue1Brown vs Liouville

So, I was watching this 3B1B video on measure theory and music (as you do) and was struck by how Grant uses epsilon to measure "how irrational" a real number is. He takes all the rationals $[0, 1]$ and assigns them progressively smaller ranges to…
2
votes
0 answers

About irrationality measure.

The irrationality measure of $\alpha\in\mathbb{R}$ is defined by $\displaystyle\mu(\alpha)=\inf\left\{\nu\in\mathbb{R}_+,\; \text{card}\left(\left\{\frac pq\in\mathbb{Q},\; 0<\left|\alpha-\frac pq\right|<\frac…
P.Fazioli
  • 223
  • 1
  • 5
2
votes
1 answer

What about irrationality of $a_n=(1-\frac12)^{(\frac12-\frac13)^{...^{(\frac{1}{n}-\frac{1}{n+1})}}}$?

This question is related to my question here which depend on the convergence of this sequence:$a_n=(1-\frac12)^{(\frac12-\frac13)^{...^{(\frac{1}{n}-\frac{1}{n+1})}}}$ however the limit of convergence is not clear even now, But my question here is…
2
votes
1 answer

Do there exist numbers with non-integer irrationality measure?

Is it possible to construct a number (by way of an infinite series or a continued fraction say) having any, possibly non-integer, irrationality measure $>2$ ? It is known that this can be done for integer irrationality measures: the Champernowne…
2
votes
0 answers

Can we make the definition of irrationality measure ‘stronger’?

By definition, $$0<\left\lvert \frac{p}{q}-x\right\rvert<\frac1{q^{\mu(x)-\epsilon}}$$ has infinitely many solutions $(p,q)$ for every $\epsilon>0$. However, to prove a theorem in a current project, I would need the following inequality to have…
Szeto
  • 10,703
  • 2
  • 14
  • 36
1
2